LSAT and Law School Admissions Forum

Get expert LSAT preparation and law school admissions advice from PowerScore Test Preparation.

User avatar
 Dave Killoran
PowerScore Staff
  • PowerScore Staff
  • Posts: 5853
  • Joined: Mar 25, 2011
|
#41330
Complete Question Explanation
(The complete setup for this game can be found here: lsat/viewtopic.php?t=8467)

The correct answer choice is (E)

This question is also suited for a Not Law attack. Since P must live one floor below K, it follows that P cannot live on the fifth floor, and therefore answer choice (E) must be correct. As you may have noticed, with many Global questions, especially the ones that appear early in a game, the first avenue of attack is to check the existing Not Laws. Make sure you always follow this guideline!
 Zarie Blackburn
PowerScore Staff
  • PowerScore Staff
  • Posts: 48
  • Joined: Jan 18, 2018
|
#65902
We recently received the following question from a student. An instructor will respond below. Thanks!
Hello,

I am studying using your text, and I believe I have come across an error in the text. A question on page 203 (#7) is explained on 205 with “E” as the correct answer. The rationale for this does not seem right based upon the setup of the question.

Could you respond to me with

1. Why this is actually correct
User avatar
 Dave Killoran
PowerScore Staff
  • PowerScore Staff
  • Posts: 5853
  • Joined: Mar 25, 2011
|
#65907
Hi, and thanks for the question!

First, (E) is the correct answer to #7, so there is NO mistake in the book. Side note: there are no known errors in the book, and it's been vetted by tens of thousands of readers.

Second, let's look at answer choice (E). Since this is a CANNOT Be True question, then it must be that P cannot live on the fifth floor for this answer to be correct. Fortunately, that means this is easy to test: if we put P on the fifth floor, then the answer will be correct if P doesn't work there. So, let's try it.

When P is placed on the fifth floor, we have the following diagram:

  • 5. P
    4.
    3.
    2.
    1.
Now, the only rule that directly involves P is the second rule, which states that, "K lives on the floor directly above P." This creates a vertical block:

  • K
    P
So, now we have a problem: with P on the fifth floor, there's no way K can live on "the floor directly above" P. Thus, when we put P on the fifth floor, we cannot create a viable solution to the game, which means that P can never go on the fifth floor. That inference is shown as a P Not Law on page 204, and discussed in the second indented point in the middle of page 205.

Answer choice (E) attempts to place P on the fifth floor, and that cannot occur. Therefore, (E) is the correct answer to this Cannot Be True question.

Thanks!

Get the most out of your LSAT Prep Plus subscription.

Analyze and track your performance with our Testing and Analytics Package.